NOMBRES - Curiosités, théorie et usages

 

Accueil                           DicoNombre            Rubriques           Nouveautés      Édition du: 05/04/2014

Orientation générale        DicoMot Math          Atlas                   Références                     M'écrire

Barre de recherche          DicoCulture              Index alphabétique                               

     

Théorème de Fermat-Wiles

 

Débutants

Triplets de Pythagore

Cas n = 3

 

Glossaire

Puissances

 

 

INDEX

 

Puissance

 

Décomposition

 

Théorème général

Familiarisation

Démonstration

Primitif (1)

Nouveau cube (2)

PGCD (3)

Équation E223  (4)

 

Sommaire de cette page

>>> Théorème pour n = 3

>>> Solution primitive

 

 

 

Théorème de Fermat-Wiles

avec n = 3 (dite E3)

Démonstration

 

Ici, vous trouverez la démonstration énoncée de façon concise. La démonstration des affirmations indiquées sont renvoyées à d'autres pages. La démonstration complète est assez longue et sa formulation linéaire sur une page ne permettrait pas de la suivre sereinement.

 

Accrochez-vous! Je vais tenter de rendre toute la démarche la plus fluide possible pour quelqu'un ayant des connaissances d'algèbre classique.

Voir Remarques liminaires sur la démonstration de E42 qui s'appliquent ici.

 

 

 

Théorème de Fermat pour n = 3

 

C'est  Leonhard Euler (1770) qui tenta de démontrer le cas n = 3 en utilisant les nombres irrationnels. Sur sa piste, Gauss, Dirichlet, et Kummer finirent la démonstration.

La preuve qui vient est plus spécifique, mais intéressante. Elle est basée sur les travaux de H. M. Edwards et présentée par Larry Freeman (en 2005).

 

 

Il s'agit de montrer que:

 

Note: nous travaillons dans l'ensemble N des nombres entiers.

 

 

x3 + y3 = z3 n'a pas de solution entière

pour x . y . z  0

{ x, y et z }  

 

Lecture des deux dernières lignes:

Pour un produit des trois nombres différent de 0, soit aucun d'eux n'est nul.

Chacun des nombres x, y et z appartient à l'ensemble N des nombres entiers.

 

 

Voici les grandes étapes.

Chacune fait l'objet d'une démonstration accessible par le lien de droite.

On part de l'hypothèse qu'il y a au moins une solution primitive (x, y et z sont premiers entre eux) sinon on divise par le facteur commun.

Solution primitive

avec z pair

et x et y impairs.

>>>

Alors, il existe p et q tels que:

 

*      PGCD(p,q) = 1;

*      p,q sont de parités opposées;

*      p,q sont positifs; et

*      2p (p² + 3q²) est un cube.

 

>>>

Nous verrons que  cette expression étant un cube alors les facteurs sont aussi des cubes.

 

p² + 3q² = u3

Une équation diophantienne dont nous aurons à trouver les solutions?

Un gros morceau! Mais, pas d'impatience, nous irons sur cette page en son temps.

 

Alors PGCD (2p, p² + 3q²)

est égal à 1 ou 3.

>>>

*    Si PGCD (2p, p² + 3q²) = 1,

alors il doit exister une solution plus petite.

>>>

*    Si PGCD (2p, p² + 3q²) = 3,

alors il doit exister une solution plus petite.

>>>

 

Dans tous les cas, une plus petite solution existe. Nous pourrions recommencer la démonstration avec cette nouvelle solution et entamer une descente infinie.

 

L'hypothèse du départ est fausse.

L'équation de Fermat pour n = 3 n'a pas de solutions.

 

 

 

 

Suite de la démonstration

*         Primitifs, pair et impairs

Voir

*         Théorème de Fermat-Wiles

*         Démonstration du cas avec la puissance quatre

Site

*         La démonstration en anglais par Larry Freeman (2005)

Autre site

*         Fermat n = 3 – ChronoMaths par Serge Mehl

Cette page

http://villemin.gerard.free.fr/Wwwgvmm/Addition/FERMAT/Fer3Intr.htm